You are filling a bookcase with books. The bookcase is 3 feet wide. Your hardcover books are 1 1/2 inches wide and your paperback books are 3⁄4 inches wide. If you have already placed 8 hardcover books on the shelf, what is the maximum number of paperback books you can also fit on the shelf?

Answers

Answer 1

Answer:

32 paperback books

Step-by-step explanation:

Given that :

Width of hardcover books = 1 1/2 inches

Width of paperback books = 3/4 inches

Number of hardcover books already on shelf = 8

Total width of shelf = 3 feets

1 foot = 12 inches

Hence, total shelf width in inches = (12 * 3) = 36 inches

Total width of hardcover books = (8 * 1 1/2) = 12 inches

Total shelf width left = (36 - 12) inches = 24 inches

Maximum number of paperback books :

Total shelf width left / paperback width

24 inches / (3/4)inches = 32 paperback books


Related Questions

HELPPPPPP please guyss

Answers

Answer:

y - 3 = 2(x - 2)

Step-by-step explanation:

Step 1. Solve for slope

m = slope

m = (y2 - y1)/(x2 - x1)

Points: (-2, 3) & (1, 9)

m = (9 - 3)/(1 - - 2)

m = 6/3

m = 2

Step 2. Use the two points and the slope to write the line in point-slope form.

Point-Slope Form: y - y1 = m(x - x1)

Slope-Intercept Form: y = 2x + 7

With the points: (-2, 3) & (1, 9)

Where x1 = -2 y1 = 3, x2 = 1 y2 = 9

Point-Slope Form = y - 3 = 2(x - 2)

*Oops, there’s something wrong with your system that there isn’t no correct option* thus the closest option would be D.

Given the table below, determine which type of equation best models the data and use a calculator to find an equation of best fit.

Answers

Answer:

Hello,

Step-by-step explanation:

Best fit: quadratic y=5.3x²-9.3+6.1

with an total error  of 0.4

A group of neighbors is holding an end of summer block party. They buy ppp packs of hot dogs, with 888 hot dogs in each pack. All together, they have 565656 hot dogs for the party.
Write an equation to describe this situation.
How many packs of hot dogs did the neighbors buy?

Answers

Answer:

565656÷8=p

So the answer is 637:D

[tex]\frac{1}{5} x + \frac{4}{5} = \frac{8}{5}[/tex]

Answers

Answer:

x = 4

Step-by-step explanation:

1/5x + 4/5 = 8/5

Multiply each side by 5 to get rid of the fractions

5(1/5x + 4/5) = 8/5 *5

x+4 = 8

Subtract 4 from each side

x+4-4 = 8-4

x = 4

Use Pythagoras to find the height and hence, the area of the triangle
below. Give height to 1 decimal place and area to the nearest whole. Write
answer in format: h= A= *
20 mm
Val

Answers

Answer:

h=17.3 A=173

Step-by-step explanation:

Calculator

Answer:

height = 17.3 mm

area = 173 mm²

Step-by-step explanation:

all three sides are of the same length (20 mm).

so, the height actually splits the baseline in half

(2 × 10 mm) while hitting it at a 90 degree angle.

so, we use Pythagoras, where the full side opposite of this 90 degree angle is c (Hypotenuse), the height of the main triangle is one side, and half of the baseline is the other side.

c² = a² + b²

20² = 10² + height²

400 = 100 + height²

300 = height²

height = 17.3 mm

the area of the main triangle is baseline (20) times height divided by 2.

so,

At = 20×17.3/2 = 10×17.3 = 173 mm²

Identify the number as prime, composite or neither. If the number is composite, write it as the product of prime
factors.
18

Answers

Answer:

composite number

prime factors={2,3}

factors of 18={1,2,3,6,9,18}

❏ Hello there! ❏

This number is composite.

Prime numbers have only two factors.

An example of a prime number is 5 (it's divisible by itself and 1)

Every prime number is divisible by itself and 1 (only these two numbers)

Composite numbers, however, have 3 or more factors.

18 has more than 3 factors, therefore, it's composite.

Product of prime factors:

2*3*3

2 is prime, because it's divisible by itself and 1

3 is prime as well

Hope it helps you!

~Just a cheerful gal

#CarryOnLearning

Answered by

[tex]-SilentNature -[/tex]

10000-10000+100000-100000

Answers

Answer:

0

Step-by-step explanation:

please help, I'm so confused how to-do this

Answers

Answer:

Step-by-step explanation:

use the quadratic formula : [tex]x = \frac{ -b \pm \sqrt{b^2 - 4ac}}{2a}[/tex], where ax^2+bx+c=0

[tex]y = \frac{ -(-20) \pm \sqrt{(-20)^2 - 4(1)(50)}}{2(1)}[/tex]

[tex]y = \frac{ 20 \pm \sqrt{400 - 200}}{2}[/tex]

[tex]y = \frac{ 20 \pm \sqrt{200}}{2}[/tex]

y = 17.07106~ or 2.928932~

PLEASE HURRY

1 Your dad asks you to go to the grocery store and buy at least 3 pounds of grapes. Write an inequality for this scenario.

Answers

Answer:

X ≥ 3

Step-by-step explanation:

Let the number of grapes you'll buy be X, you need to buy at leaF 3 pounds so:

X ≥ 3

Answered by Gauthmath

The average age of a preschool class is 4.5 years old. If there is one 3-year-old, five 5-year-olds, and two other children both of the same age, what is the age, in years, of the other two children?

Answers

Answer:

3.1

Step-by-step explanation:

5 x5 =25

25+3=28

the other children are 4 years old

martin is filling his bathtub, but he left the drain partially open. martin knows that it takes 8 minutes to fill his 40 gallon tub. the equation v = 2.5t represents the volume, v , of water that drains out of the tub in t minutes. if martin leaves the water on, will the tub ever overflow ?

please someone help ​​

Answers

Answer:

Yes, it will overflow after 16 minutes..

Step-by-step explanation:

The tub fills at a rate of 5 gallons every minute, but it drains at a rate of 2.5 gallons every minute. This means that it fills twice as fast as it drains, so it would take 16 minutes for the tub to overflow. If you subtract the rate of draining from the rate of filling then the net rate of water going up is 2.5 gallons a minute.

if three less than one half a number is equal to one-third of the same number, find the number​

Answers

Answer: The number would be 18

Step-by-step explanation:

18/2=9 9-3=6 6*3=18

The equation be (x/2) - 3 = x/3 then, the number of x = 18.

How to estimate the value of x?

To estimate the value of x, bring the variable to the left side and bring all the remaining values to the right side. Simplify the values to estimate the result.

Let x be the number

From the given information, we get

(x/2) - 3 = x/3

The set all the fractions on one side and constants on another side

(x/2) - (x/3) = 3

Multiply by LCM

3x - 18 = 2x

Add 18 to both sides

3x - 18 + 18 = 2x + 18

simplify

3x = 2x + 18

Subtract 2x from both sides

3x - 2x = 2x + 18 - 2x

x =18.

Therefore, the number is x = 18.

To learn more about algebraic expression refer to:

https://brainly.com/question/4541471

#SPJ2


Describe fully the single transformation that maps shape A onto shape B.

Answers

B is an upside down image of A and is 2 units below A.

The image of A is mirrored across the y axis at y = 2.

Answer: A was mirrored across y= 2 to form B.

please help with number 9!!!

Answers

Answer:

2 + sqrt(3)

Step-by-step explanation:

Roots that contain square roots come in pairs

If there is a root that is a- sqrt(b), there is a root that is a+ sqrt(b)

2 -sqrt(3) means there is a root 2 + sqrt(3)

The width of a rectangle measures (2h-6) centimeters, and its length measures (7h+8) centimeters. Which expression represents the perimeter, in centimeters, of the rectangle?

Answers

Answer:

18h +4

Step-by-step explanation:

The perimeter of a rectangle is given by

P = 2(l+w)  where l is the length and w is the width

  =2( 7h+8 + 2h-6)

Combine like terms

  = 2( 9h+2)

Distribute

   = 18h +4

Find the missing side lengths

Answers

Answer:

u is 172 and v is 17

Step-by-step explanation:

To find u:

[tex]{ \bf{ \sin( \theta) = \frac{opposite}{hypotenuse} }}[/tex]

feed in the terms:

[tex] \sin(45 \degree) = \frac{17}{u} \\ \\ u = \frac{17}{ \sin(45 \degree) } \\ \\ u = 17 \sqrt{2} [/tex]

To find v:

[tex] \cos( \theta) = \frac{adjacent}{hypotenuse} [/tex]

feed in the terms:

[tex] \cos(45) = \frac{v}{u} \\ \\ v = u. \cos(45) \\ v = (17 \sqrt{2} )÷( \sqrt{2} ) \\ v = 17[/tex]

Find the principle that yields and interest of Rs.2240 at the rate of 10% p.a. in a years 6 months.​

Answers

Step-by-step explanation:

I=PRT/100

p=I/RT×100

p=2240/10×2×100

p=11200

The sets A and L are given below.
A = (a, b, e}
L={c, k,j}
Find the intersection of A and L.
Find the union of A and L.
Write your answer using notation (in roster form)
Help plzzz

Answers

Step-by-step explanation:

A n L =null

A U L= a,b,c,e,k,j

Selling frozen yogurt at a fair, you make $565 and use 250 cones. A single-scoop cone cost $2 and a double-scoop cone cost $2.50. How many of each type of cone did you sell?

Answers

The seller sold 120 single-scoop cones and 130 double-scoop cones to make $565.

250 cones were used while selling yogurt and made $565.

There are two types of cones:

Single-scoop cone that cost $2.

Double-scoop cone that cost $2.50.

We need to know how many cones of each type were used while selling yogurt that made $565.

This is a system of linear equations problem.

We will have to make two linear equations using the given statements and apply the substitution method to get our answer.

Consider,

x = single-scoop cone.

y = double-scoop cone.

Now, we can make two linear equations:

x + y = 250............(A)

2x + 2.50y = 565...............(B)

Apply substitution method.

From (A) we get,

x = 250 - y...............(C)

And putting x = 250 - y in (B) we get,

2 ( 250 - y ) + 2.50y = 565

500 - 2y + 2.50y = 565

0.50y = 565 - 500

0.50y = 65

y = 65 / 0.50
y = 6500 / 50

y = 130

Putting y = 50 in (C) we get,

x = 250 - 130

x = 120

We have,

x = 120 and y = 130

We see that we need 120 single-scoop cones and 130 double-scoop cones to make $565.

Learn more about the system of the linear equations here:

https://brainly.com/question/8062932

#SPJ2

please help me with this

Answers

Answer:

a) 10

b) 2.5

Step-by-step explanation:

2^n = 1024

2^n = 2^10

n = 10

4^(2n-3) = 16

4^(2n-3) = 4^2

2n-3 = 2

2n = 5

n = 2.5

Solution:-1:-

[tex]\\ \sf\longmapsto 2^n=1024[/tex]

Equal the bases

[tex]\\ \sf\longmapsto 2^n=2^10[/tex]

[tex]\\ \sf\longmapsto n=10[/tex]

Solution:-2:-

[tex]\\ \sf\longmapsto 4^{2n-3}=16[/tex]

[tex]\\ \sf\longmapsto 4^{2n-3}=4^2[/tex]

[tex]\\ \sf\longmapsto 2n-3=2[/tex]

[tex]\\ \sf\longmapsto 2n=2+3[/tex]

[tex]\\ \sf\longmapsto 2n=5[/tex]

[tex]\\ \sf\longmapsto n=\dfrac{5}{2}[/tex]

[tex]\\ \sf\longmapsto n=2.5[/tex]

what is the external angle of a polygon where the corresponding interior angle equals 105 degrees

Answers

Answer:

75 degrees maybe.......

Answer:

75 degrees

Step-by-step explanation:

the external angle is between the outside of one of the sides of the angle and the continued line of the second side of the angle.

and because it is measured against a line, where we have a total of 180 degrees for angles, we have

exterior angle = 180 - interval angle = 180-105 = 75

If i work in a week 2 days cleaning and i will work 4 hours in a day and in 1 hour is 8$ how much will it be in a year

Answers

Answer:

$3072

Step-by-step explanation:

It is given that :

I work for 4 hours everyday, working 2 days a week.

And in 1 hour , I get $8.

Therefore,

1 hour = $8

4 hours = 4 x  8

             = $32

So, in 1 day working for 4 hours, I get $32.

∴ Working 2 days = 32 x 2

                              = $64

In 1 month, there are 4 weeks

So, in 4 weeks (or 1 month), I work for = 4 x 2

                                                               =  8 days

Therefore, in 8 days, I get = 8 x 32

                                            = $256

Now there are 12 months in a year.

So, in 12 months , I will get = 12 x 256

                                            = $3072

Therefore, in 1 year , I will get $3072.

Geometry, please answer question ASAP

Answers

9514 1404 393

Answer:

  C.  16

Step-by-step explanation:

UV is a diameter of the circle. If it is perpendicular to the chord XS, then it bisects that chord. This means ...

  XT = TS

  3b -1 = b +5

  2b = 6

  b = 3

  TS = b +5 = 8

The chord length is 2·TS = 2·8 = 16 units.

Answer:

C,XS=16

Step-by-step explanation:

3b-1=b+5

3b-b=5+1

2b=6

b=6/2=3

TS=b+5=3+5=8

XS=2 TS=2×8=16

sinx + cos2x + sin3x + 1 =0

Answers

Step-by-step explanation:

This is true statement.

have a great day

The distance between points A(0,1) and B(x, 4) is p

34. Find the x coordinate for point B.

Answers

Answer:

X = (P^2 - 9) ^ 1/2

Step-by-step explanation:

P^2 = (4 - 1)^2+ (0 -X)^1/2

If the angle is in standard position and P(x, y) is a point on the terminal side of , and r is the distance from the origin to P, then sin() = Incorrect: Your answer is incorrect. cos() = tan() = .

Answers

cos(a) = x/r

sin(a) = y/r

tan(a) =  y/x

In the image at the end, you can see a sketch of the situation, we can model this with a triangle rectangle:

The hypotenuse length is equal to r, the adjacent cathetus to the angle is the x-value, and the opposite cathetus to the angle is the y-value.

Now we can remember the trigonometric relations:

cos(a) = (adjacent cathetus)/(hypotenuse)

sin(a) = (opposite cathetus)/(hypotenuse)

tan(a) = (opposite cathetus)/(adjacent cathetus)

Using what we wrote above, we can rewrite these as:

cos(a) = (adjacent cathetus)/(hypotenuse) = x/r

sin(a) = (opposite cathetus)/(hypotenuse) = y/r

tan(a) = (opposite cathetus)/(adjacent cathetus) = y/x

if you want to learn more about this topic, you can read:

https://brainly.com/question/24297646

18) Mala gave Kamala half the number of sweets she had. She then had 7 sweets left. How many sweets did Mala have at the start?​

Answers

Answer:

Mala had 14 sweets at the start.

Step-by-step explanation:

7 x 2 = 14

To check, 14/2 = 7. (Dividing by 2 is the same as multiplying by 1/2)

Correct answer gets brainliest and 5 star

Answers

Answer:

d

Step-by-step explanation:

Answer:

option D

Step-by-step explanation:

the formula for slope is: y = mx + b

where m = slope & b = y intercept

so in y = -2x + 1,

m (slope) = -2     &      b (y-int) = (0,1)

Write an equivalent expression for each of the following. Express your answers in the form a + bi or a - bi

(5 + i)(9 + i)

Help please

Answers

Answer:

44 +14i

Step-by-step explanation:

(5 + i)(9 + i)

FOIL

first 5*9 = 45

inner: 9i

outer :5i

last: i*i = i^2 = -1

Add togheter

45 + 9i+5i -1

Combine like terms

44 +14i

What is the value of g? ​

Answers

Answer:

56 degrees

Step-by-step explanation:

1. Notice how g is part of a right angle, which equals 90 degrees.

2. Notice how the 34 degree angle on the other side is also part of a 90 angle.

3. Notice how the 2 right angles are vertical to each other, meaning they are the same.

4. Subtract 34 from 90.

5. g=56 degrees

Hope this helps!

-Stella

I think it would be 90 degrees. The g marking looks like it's taking up 2 angles. One of the angles is 34 degrees, because it's opposite to the one marked. Together they look like they form a 90 degree angle.

*It's kind of of hard to see if g is referring to both angle degrees or not.

Other Questions
Need help with this one too please thank you. Someone help pleasee DUDE I NEEED HELP FRIST ANSWER GET BRIANLY Rachel has 37 videos and decides to purchase 2 more each week. Write an equation describing this situation. The British won the French and Indian War and received lands and colonies in their victory. Of the following choices, which colony WAS NOT given to the British? 2. Puede haber trabajo en un sistema si no hay movimientoa. S, si hay una fuerza siempre. b. S, ya que el movimiento es slo relativo. c. No, ya que un sistema que no se mueve no tiene energa. d. No, porque la ecuacin de trabajo lo define. For centuries scientists found evidence that supports the theory of spontaneous generation. Which of the followingsupported this theory?A. The sun rising in the east and seeing in the west.B. Maggots appearing on rotten meat.C. The swan-next flask experiment.D. All living things are made of cells. Refer to picture Need help on Q.8 help please ill give brainliest is placing a compass near a wire with live electrical current biology, chemistry, or physics? Suppose the volume of the cone is 324pi Find dy/dx when x=6 and y=27 Gabriella has a smart phone data plan that costs $25 per month that includes 10 GB of data, but will charge an extra $15 per GB over the included amount. How much would Gabriella have to pay in a month where she used 5 GB over the limit? How much would Gabriella have to pay in a month where she used went over by x x GB? find the next three sequence of 1,3, 9, 27, team skills do not require the ability to communicate effectively.true or false Which statement accurately contrasts the Ottoman and Safavid empire in the 16 century Plz answer asap question in picture PLEASE HELP THIS IS DUE ASAP (answer in decimal!!!!) Given the formula A = 5h (B + b); solve for B.2 "If a nation is rich in resources then it can be developed! Justify this statement. In the Middle East, the United States and the Soviet Union both had an interest in:-oil reserves.-cheap labor.-religiously significant sites.-sources of drinkable water.